You are on page 1of 4

Please let me know if you nd any mistakes. 1.

For each natural number n and each number x [0, 1], dene fn (x) = x xn . (a) Find the function f : [0, 1] R to which the sequence {fn : [0, 1] R} converges pointwise. Proof: For x = 0, fn (0) = 0 0n = 0 for all n. Thus f (0) = 0. Similarly for x = 1, fn (1) = 1 1n = 0 for all n. So f (1) = 0. Now for x (0, 1), lim fn (x) = lim (x xn ) = x lim xn = x 0 = x. Therefore
n n n

f (x) =

x if x [0, 1) 0 if x = 1.

Note: Although these limits are clear, if asked to, you should be able to prove them using the denition of limits. (b) Prove or disprove that the convergence is uniform. Proof: Suppose that {fn (x)} converges to f (x) uniformly on [0, 1]. Since each fn (x) is a polynomial, it is continuous. Also by a theorem (Theorem 9.31), the uniform limit of a continuous functions is continuous. Thus f (x) is continuous 1 on [0,1]. Let xn = 1 n . Then xn [0, 1] for all n N and {xn } 1. Also 1 f (xn ) = xn = 1 n , so lim f (xn ) = 1 = f (1). Which contradicts that f (x)
n

is continuous on [0.1].

2. Suppose converges.
k=0

ak converges and that ak 0 for all k N. Let p > 1. Prove that


k=0

(ak )p

Proof: Since
k=0

ak converges, there exists M > 0 such that


k=0 k

ak < M . Moreover

we know lim ak = 0. Then there exists an N N such that k N implies ak < 1. Then for k N , (ak )p < ak . Since ak 0 for all k, we have
n k=N

(ak )p <

ak < M . Let sn =
k=N k=0

(ak )p . Then sn M + sN for all n. Hence {sn } is

bounded above. Moreover, since ak 0, sn+1 sn for all n, so the sequence is (ak )p converges.
k=0 1 n

monotonically increasing. Therefore {sn } converges, so

3. For each natural number n and each number x R, dene fn (x) =

sin(nx).

(a) Show that {fn } converges uniformly to the function f (x) = 0 x R. Proof: Let > 0 be given. Then by the Archimedean Property there exists an N N such that n > N implies 1/n < . Then for n > N , |fn (x) f (x)| = 1 1 1 sin(nx) 0 = n sin(nx) n < for all x R. n

(b) Notice that each fn is dierentiable everywhere (Do not prove this). Show that the pointwise limit of {fn (x)} need not exist and explain why this does not contradict the theorem regarding uniformly convergent sequences of dierentiable functions (Theorem 9.33). Proof: Notice that fn (x) = cos(nx). Let x = . Then fn () = 1 if n is even and fn () = 1 if n is odd. Thus the sequence {fn ()} does not converge. Note: As with the rst problem, you should be able to prove using the denition of a limit that this sequence does not converge. 4. Let f and g be continuous real valued functions on (a, b) such that f (r) = g(r) for each r Q (a, b). Prove that f (x) = g(x) for all x (a, b). Proof Dene h(x) = f (x) g(x). Then h(x) is the dierence of two continuous functions and therefore is continuous. Thus it suces to show that h(x) = 0 for all x (a, b). Suppose that there exists an x (a, b) such that h(x) = 0. Then by the density of the rational numbers there exists a sequence {rn } (a, b) Q such that {rn } x. Then by continuity {h(rn )} h(x). Notice that h(rn ) = 0 for all n. Therefore {h(rn )} 0 = h(x), which is a contradiction. 5. Prove that | sin(x)sin(y)| |xy| for all x, y R. (Hint: use Mean Value Theorem). Then use this and the denition of uniform continuity to prove that sin(x) is uniformly continuous on R. Proof: Fix x, y R (WLOG x < y). Let f (t) = sin(t). Then f (t) = cos(t). Since f is continuous on [x, y] and f is dierentiable on (x, y), by the Mean Value Theorem there exists an x0 (x, y) such that f (x)f (y) = f (x0 )(xy). Since | cos(t)| 1 for all t we have | sin(x) sin(y)| = | cos(x0 )(x y)| |x y|. Now let > 0 be given. Choose = . Then |u v| < implies | sin(u) sin(v)| |u v| < = for all u, v R. So sin(x) is uniformly continuous on R.

6. Prove that if verges.


n=0

|an | converges and {bn } is a bounded sequence, then


n=0

an bn con-

Hint: Let {sn } and {tn } be the sequence of partial sums of


j=0

|aj | and
j=0

aj bj respec-

tively. Since {bn } is a bounded sequence, there exists M > 0 such that |bn | < M for all n N. Also since
n=0

|an | converges, the sequence {sn } is Cauchy Let > 0


M

be given. Then there exists an N N such that n > N implies |sn+k sn | <

for all k N. Then for n > N ,


n+k n

|tn+k tn | =
j=0

aj b j
j=0

aj b j

= |an+k bn+k + .... + an+2 bn+2 + an+1 bn+1 | |an+k bn+k | + .... + |an+2 bn+2 | + |an+1 bn+1 | M (|an+k | + .... + |an+2 | + |an+1 |) = M |sn+k sn | <M M = .
j=0

Therefore {tn } is Cauchy and hence

aj bj converges.

7. Let {an } be a Cauchy sequence. Suppose {an } has a convergent subsequence {ank } converging to a number a. Prove using the denition of convergence that {an } a. Proof: Let > 0 be given. Since {an } is Cauchy, there exists an N N such that |an am | < 2 for n, m > N . Also, since {ank } a, there exists a K N such that k > K implies |ank a| < 2 . Fix nk such that k > K and nk > N . Then n > N implies |an a| = |an ank + ank a| |an ank | + |ank a| Thus {an } a. 8. Suppose that the function f : R R has the property that x2 f (x) x2 , x R. 2 + 2 = .

Prove that f is dierentiable at x = 0 and that f (0) = 0. Proof: Notice since x2 f (x) x2 , |f (x)| x2 for all x R. Then f (0) = 0. Let {xn } be a sequence in R\{0} converging to 0. Then given > 0, there exists an N N such that n > N implies |xn | < . Now for n > N f (xn ) f (0) f (xn ) 0 0 = xn 0 xn f (xn ) = xn (xn )2 xn = |xn | < . Therefore lim f (x) f (0) = 0, so f is dierentiable at 0 and f (0) = 0. x0 x0

9. Suppose that the function f : [a, b] R is dened by f (x) = c for all x (a, b] and f (a) = d R. Use the denition of the integral, the Archimedes Riemann Theorem, or the Cauchy Criterion for integrals to show that f is integrable on [a, b]
b

and
a

f = c(b a).

1 Proof: Dene a sequence of partitions, Pn of [a, b] where Pn = {a, a + n , b}. Let M = max{c, d} and m = min{c, d}. Then

lim (U (f, Pn ) L(f, Pn )) = lim (M

1 1 1 1 + c(b (a + ))) (m + c(b (a + ))) n n n n n 1 = lim (M m) = 0. n n


b

. Thus by the Archimedes-Riemann Theorem, f is integrable. Lastly,


n

f =
a

lim U (f, Pn ) = lim (M


n

1 1 + c(b (a + ))) = c(b a). n n

You might also like